12x-3.7+2x+13.3+4.1y simplified just give me the answer i dont really need an explanation. Thanks

Answers

Answer 1

The simplified expression is

14x + 9.6 + 4.1y

What is an expression?

An expression is a way of writing a statement with more than two variables or numbers with operations such as addition, subtraction, multiplication, and division.

Example: 2 + 3x + 4y = 7 is an expression.

We have,

12x -3.7 + 2x + 13.3 + 4.1y

Combining the like terms.

(12x + 2x) - 3.7 + 13.3 + 4.1y

14x + 9.6 + 4.1y

Thus,

The final expression is 14x + 9.6 + 4.1y.

Learn more about expressions here:

https://brainly.com/question/3118662

#SPJ1


Related Questions

Kitchenaid will discontinue the bisque color for its dishwashers due to reports suggesting it is not popular west of the Mississippi unless more than 30% of its customers in states east of the Mississippi prefer it to make up for lost sales elsewhere). As part of the decision process, a random sample of 500 customers east of the Mississippi is selected and their preferences are recorded. of the 500 interviewed, 185 said they prefer the bisque color. a. (3 pts) Define the parameter of interest in words and notation. b. (4 pts) state the null and alternative hypotheses in words with context. C. (2 pts) Let's perform the hypothesis test nonparametrically. Complete the code below to simulate data under the assumption of the null hypothesis by replacing the question marks with the appropriate number of simulations and the appropriate number of observations to resample. Give the histogram of the null distribution.

Answers

a. The parameter of interest is the proportion of Kitchenaid dishwasher customers east of the Mississippi who prefer the bisque color (p).

b. Null hypothesis: The proportion of customers east of the Mississippi who prefer the bisque color is less than or equal to 0.3 (p <= 0.3); Alternative hypothesis: The proportion of customers east of the Mississippi who prefer the bisque color is greater than 0.3 (p > 0.3).

a. The parameter of interest is the proportion of Kitchenaid dishwasher customers east of the Mississippi who prefer the bisque color. It can be denoted as p.

b. The null hypothesis is that the proportion of customers east of the Mississippi who prefer the bisque color is less than or equal to 0.3, i.e., p <= 0.3. The alternative hypothesis is that the proportion of customers east of the Mississippi who prefer the bisque color is greater than 0.3, i.e., p > 0.3. This is based on the condition that if less than 30% of customers east of the Mississippi prefer the bisque color, then the color will be discontinued unless more than 30% of its customers in states east of the Mississippi prefer it to make up for lost sales elsewhere.

c.

# set the random seed for reproducibility

set.seed(1234)

# number of simulations

num_sims <- ???

# number of observations to resample

sample_size <- ???

# vector to store the simulated proportions

sim_props <- numeric(num_sims)

# simulate the null hypothesis

for (i in 1:num_sims) {

 # randomly sample from a population with p = 0.3

 sample_data <- sample(c("bisque", "other"), size = sample_size, replace = TRUE, prob = c(0.3, 0.7))

 # calculate the proportion who prefer bisque

 sim_props[i] <- sum(sample_data == "bisque") / sample_size

}

# plot the histogram of the null distribution

hist(sim_props, breaks = 20, col = "gray", main = "Null Distribution", xlab = "Proportion")

Note: In the code above, we simulate the null hypothesis by randomly sampling from a population with a proportion of 0.3 who prefer the bisque color, and 0.7 who prefer other colors. We simulate this process for a specified number of simulations (denoted as "num_sims") and for a specified sample size (denoted as "sample_size"). The resulting proportions are stored in a vector called "sim_props". We then plot the histogram of the null distribution using the hist() function.

To learn more about null hypothesis visit : https://brainly.com/question/4436370

#SPJ11

If you put 90 ml of concentrate in a glass how much water should be added

Answers

If you put 90 ml of concentrate in a glass, you should add 210 ml of water to dilute it to a 1:3 concentration ratio.

To understand why, we need to use the concentration ratio formula, which is:Concentration Ratio = Concentrate Volume / Total VolumeWe can rearrange the formula to solve for the Total Volume:Total Volume = Concentrate Volume / Concentration RatioIn this case, we know the Concentrate Volume is 90 ml, but we don't know the Concentration Ratio. However, we know that the ratio of concentrate to water should be 1:3. This means that for every 1 part of concentrate, we should have 3 parts of water. This gives us a total of 4 parts (1+3=4). Therefore, the Concentration Ratio is 1/4 or 0.25.To find the Total Volume, we can substitute the known values:Total Volume = 90 ml / 0.25 = 360 mlThis is the total volume of the mixture if we were to use a 1:3 concentration ratio.

However, the question asks how much water should be added. So, to find the amount of water, we need to subtract the concentrate volume from the total volume:Water Volume = Total Volume - Concentrate VolumeWater Volume = 360 ml - 90 mlWater Volume = 270 mlTherefore, you should add 270 ml of water to 90 ml of concentrate to dilute it to a 1:3 concentration ratio.

Learn more about Concentration ratio here,A concentration ratio indicates the:

a. number of firms in an industry.

b. number of large firms in an industry compa...

https://brainly.com/question/15848538

#SPJ11

Given a standard normal distribution, find the value of k such that (a) P(Z > k) = 0.2046: (b) P(Z < k) = 0.0427: (c) P(-0.93 < Z < k) = 0.7235.

Answers

The value of k for part (c) is 0.15.

(a) To find the value of k such that P(Z > k) = 0.2046, we need to look up the z-score that corresponds to a cumulative probability of 1 - 0.2046 = 0.7954. Using a standard normal table or a calculator, we can find that the z-score for this probability is approximately 0.84. Therefore, k = -0.84.

(b) Similarly, to find the value of k such that P(Z < k) = 0.0427, we need to look up the z-score that corresponds to a cumulative probability of 0.0427. Using a standard normal table or a calculator, we can find that the z-score for this probability is approximately -1.71. Therefore, k = -1.71.

(c) To find the value of k such that P(-0.93 < Z < k) = 0.7235, we need to first find the z-score that corresponds to a cumulative probability of (1 - 0.7235)/2 = 0.13825, which is the probability to the left of -0.93. Using a standard normal table or a calculator, we can find that the z-score for this probability is approximately -1.08.

Then, we need to find the z-score that corresponds to a cumulative probability of 1 - 0.13825 = 0.86175, which is the probability to the right of k. Using a standard normal table or a calculator, we can find that the z-score for this probability is approximately 1.08.

The value of k can be found by adding the z-scores for the probabilities to the left and right of k: k = -0.93 + 1.08 = 0.15. Hence, the value of k for part (c) is 0.15.

To know more about probability refer to-

https://brainly.com/question/30034780

#SPJ11

(1 point) find the area of the region outside r=5 5sinθ , but inside r=15sinθ.

Answers

The area of the region outside r=5sinθ and inside r=15sinθ is 50π square units.

The area of the region outside r=5sinθ and inside r=15sinθ, we need to evaluate the integral of the area element dA over the region of interest. The area element in polar coordinates is given by dA = r dr dθ.

The region of interest is the annular region between the two circles, which is defined by the inequalities:

5sinθ ≤ r ≤ 15sinθ

0 ≤ θ ≤ π

Thus, the area of the region is given by:

A = [tex]\int\int dA = \int_0^\pi \int_5sin\theta^{(15sin\theta)} r dr d\theta[/tex]

Using the limits of integration, we can rewrite the integral as:

A = [tex]\int_0^\pi [1/2 (15sin\theta)^2 - 1/2 (5sin\theta)^2] d\theta[/tex]

Simplifying the integrand, we get:

A = [tex]1/2 \int_0^\pi (225sin^2\theta - 25sin^2\theta) d\theta[/tex]

A = [tex]1/2 \int_0^\pi 200sin^2\theta d\theta[/tex]

Using the identity sin²θ = 1/2 - 1/2cos2θ, we get:

A =[tex]1/2 \int_0^\pi 100 - 100cos2\theta d\theta[/tex]

Integrating, we get:

A = 1/2 [100θ - 50sin2θ] from 0 to π

A = 1/2 [100π - 0] - 1/2 [0 - 0]

A = 50π

For similar questions on area

https://brainly.com/question/25292087

#SPJ11

The area of the region outside the polar curve r = 5sinθ and inside the polar curve r = 15sinθ is 50π square units.

To calculate the area between two polar curves, we integrate the outer curve and subtract the integral of the inner curve over the desired interval. In this case, the curves are r = 5sinθ and r = 15sinθ, and we want to find the area from θ = 0 to θ = π.

The equation r = 5sinθ represents the inner curve, and r = 15sinθ represents the outer curve.

Using the formula for the area between two polar curves, the area A can be calculated as follows:

A = (1/2) ∫[θ1,θ2] (r_outer^2 - r_inner^2) dθ

Substituting the given equations, we have:

A = (1/2) ∫[0,π] ((15sinθ)^2 - (5sinθ)^2) dθ

Simplifying the equation further:

A = (1/2) ∫[0,π] (225sin^2θ - 25sin^2θ) dθ

A = (1/2) ∫[0,π] 200sin^2θ dθ

Integrating this equation over the given interval, we get:

A = (1/2) * 200 * ∫[0,π] sin^2θ dθ

Using the identity ∫ sin^2θ dθ = (1/2) * (θ - sinθcosθ), we have:

A = (1/2) * 200 * [(π - sinπcosπ) - (0 - sin0cos0)]

A = (1/2) * 200 * [(π - 0) - (0 - 0)]

A = (1/2) * 200 * π

A = 100π

Finally, we subtract the area enclosed by the inner curve r = 5sinθ to get the area between the curves:

A = 100π - (1/2) * 5^2 * π

A = 100π - 25π

A = 75π

Therefore, the area of the region outside r = 5sinθ but inside r = 15sinθ is 50π square units.

To learn more about polar curves click here

brainly.com/question/27548409

#SPJ11

Sal's pet store only sells lizards and birds. Sal currently has 16 birds and 18 lizards available for sale. Six of
the birds and 14 of the lizards are male. What is the probability that a randomly selected pet is a lizard given that it is a female?

Answers

Answer:

  d)  2/7

Step-by-step explanation:

You want the probability that a pet is a lizard, given that it is female if 14 of 18 lizards are male, and 6 of 16 birds are male.

Female

There are 10 female birds and 4 female lizards, so 4 of (10+4) = 14 female pets are lizards.

  P(lizard | female) = 4/14 = 2/7 . . . . matches choice D

<95141404393>

The actual diameter of uranus is 31,250 miles. in a scale drawing of the solar system, the diameter of uranus is 125 centimeters.

what scale was used to make the model?

Answers

In the scale drawing of the solar system, the scale used to represent Uranus with a diameter of 31,250 miles as 125 centimeters is 1 centimeter representing 250 miles.

To determine the scale used in the model, we can establish a ratio between the actual diameter of Uranus and its representation in the scale drawing.

The actual diameter of Uranus is 31,250 miles, while its representation in the scale drawing is 125 centimeters. Let's assume the scale is represented as 1 centimeter representing "x" miles. We can set up a proportion:

1 centimeter / x miles = 125 centimeters / 31,250 miles

Cross-multiplying gives us:

1 * 31,250 = 125 * x

31,250 = 125x

Dividing both sides by 125, we find:

x = 31,250 / 125

x = 250

Therefore, the scale used in the model is 1 centimeter representing 250 miles. This means that each centimeter in the scale drawing corresponds to 250 miles in reality. In other words, the diameter of Uranus is scaled down by a factor of 250. So, if we measure 1 centimeter in the model, it would represent a distance of 250 miles in the actual solar system.

Learn more about Cross-multiplying here:

https://brainly.com/question/28308012

#SPJ11

show that if A has n linearly independent eigenvectors, then so does A^T. If A has n linear independent eigenvectors, complete the statements below based on the Diagonalization Theorem. A can be factored as ____ The ____ of matrix P are n linearly independent ______
D is a diagonal matrix whose diagonal entries are_____

Answers

A can be factored as [tex]A = PDP^{(-1)}[/tex]

The columns of matrix P are n linearly independent eigenvectors.

D is a diagonal matrix whose diagonal entries are the eigenvalues corresponding to the eigenvectors in P.

To show that if matrix A has n linearly independent eigenvectors, then so does its transpose [tex]A^T[/tex], we can use the following argument:

Let [tex]v_1, v_2, ..., v_n[/tex] be n linearly independent eigenvectors of A corresponding to eigenvalues [tex]λ_1, λ_2, ..., λ_n,[/tex] respectively. Then, by definition, we have:

[tex]A v_1 = λ_1 v_1 \\ A v_2 = λ_2 v_2 \\ A v_n = λ_n v_n[/tex]

Taking the transpose of both sides of these equations, we get:

[tex](A v_1)^T = (λ_1 v_1)^T \\ v_1^T A^T = λ_1 v_1^T[/tex]

Similarly,

[tex]v_2^T A^T = λ_2 v_2^T\\ v_n^T A^T = λ_n v_n^T[/tex]

Now, let's examine the equations

[tex]v_1^T A^T = λ_1 v_1^T \: and \: v_2^T A^T = λ_2 v_2^T[/tex]

. If we subtract [tex]λ_1[/tex] times the first equation from [tex]λ_2[/tex] times the second equation, we get:

[tex]v_2^T A^T - λ_2 v_1^T A^T = λ_2 v_2^T - λ_1 λ_2 v_1^T \\ (v_2^T - λ_1 v_1^T) A^T = (λ_2 - λ_1 λ_2) v_2^T[/tex]

Notice that [tex]v_2^T - λ_1 v_1^T[/tex] is a non-zero vector because [tex]v_1 \: and \: v_2[/tex] are linearly independent. Therefore, for the equation above to hold [tex]A^T[/tex]

must have an eigenvector corresponding to the eigenvalue [tex](λ_2 - λ_1 λ_2)[/tex]

By repeating this process for all pairs of eigenvectors [tex](v_i, v_j)[/tex] and eigenvalues [tex](λ_i, λ_j)[/tex], we can see that [tex]A^T[/tex] has at least n linearly independent eigenvectors corresponding to its eigenvalues.

Now, based on the Diagonalization Theorem, if A has n linearly independent eigenvectors, it can be factored as:

[tex]A = PDP^{(-1)}[/tex] Where P is a matrix whose columns are the n linearly independent eigenvectors of A, and D is a diagonal matrix whose diagonal entries are the corresponding eigenvalues.

Therefore, we can complete the statements as follows:

A can be factored as [tex]A = PDP^{(-1)}[/tex]

The columns of matrix P are n linearly independent eigenvectors.

D is a diagonal matrix whose diagonal entries are the eigenvalues corresponding to the eigenvectors in P.

Learn more about matrix here,

https://brainly.com/question/31864533

#SPJ4

Maximize Z = 4X1 + 5X2
Subject to:
X1 + 2X2 ≤ 10
6X1 + 6X2 ≤ 36
X1 ≤ 4
X1, X2≥ 0.
Based on the linear programming model shown above, answer the following questions:
A: How many decision variables does this problem have? What are they?
B: Apart from the non-negativity constraints, how many other constraints does this problem have? What are they?
C: What is the optimal solution value for X1 and X2 in this linear programming model? Use the linear programming module in the POM software downloads to the value for X1 and X2. The link for this software download is given below.

Answers

a) [tex]X_{1}[/tex] and [tex]X_{2}[/tex] are the two choice variables in this issue.

b) In addition to the non-negativity criteria, this problem contains three further constraints.

c) The optimal solution for this linear programming model is [tex]X_{1}[/tex] = 4 and  [tex]X_{2}[/tex]  = 3, with an optimal objective function value of 28.

A) This problem has two decision variables,  [tex]X_{1}[/tex]  and  [tex]X_{2}[/tex] .

B) This problem has three other constraints apart from the non-negativity constraints. They are:

[tex]X_{1}[/tex]  + 2[tex]X_{2}[/tex] ≤ 10

6[tex]X_{1}[/tex] + 6[tex]X_{2}[/tex] ≤ 36

[tex]X_{1}[/tex]  ≤ 4

C) To find the optimal solution value for  [tex]X_{1}[/tex]  and  [tex]X_{2}[/tex] , we need to solve the linear programming model using a suitable method such as the simplex method. Solving the problem using the simplex method, we get the optimal solution as:

[tex]X_{1}[/tex]  = 4,  [tex]X_{2}[/tex]  = 3

The optimal objective function value is Z = 4 [tex]X_{1}[/tex]  + 5[tex]X_{2}[/tex] = 4(4) + 5(3) = 28. Therefore, the optimal solution for this linear programming model is [tex]X_{1}[/tex] = 4 and  [tex]X_{2}[/tex] = 3, with an optimal objective function value of 28.

To learn more about variables here:

https://brainly.com/question/31432068

#SPJ4

In this problem, we use your critical values table to explore the significance of r based on different sample sizes. (a) Is a sample correlation coefficient rho = 0.82 significant at the α = 0.01 level based on a sample size of n = 3 data pairs? What about n = 14 data pairs? (Select all that apply.) No, because the absolute value of the given correlation coefficient is smaller than that for a sample size of n = 14 and α = 0.01. No, because the absolute value of the given correlation coefficient is greater than or equal to that for a sample size of n = 3 and α = 0.01. Yes, because the absolute value of the given correlation coefficient is greater than or equal to that for a sample size of n = 3 and α = 0.01. Yes, because the absolute value of the given correlation coefficient is smaller than that for a sample size of n = 3 and α = 0.01. Yes, because the absolute value of the given correlation coefficient is smaller than that for a sample size of n = 14 and α = 0.01. No, because the absolute value of the given correlation coefficient is smaller than that for a sample size of n = 3 and α = 0.01. Yes, because the absolute value of the given correlation coefficient is greater than or equal to that for a sample size of n = 14 and α = 0.01. No, because the absolute value of the given correlation coefficient is greater than or equal to that for a sample size of n = 14 and α = 0.01. Incorrect: Your answer is incorrect. (b) Is a sample correlation coefficient rho = 0.42 significant at the α = 0.05 level based on a sample size of n = 18 data pairs? What about n = 26 data pairs? (Select all that apply.) Yes, because the absolute value of the given correlation coefficient is greater than or equal to that for a sample size of n = 26 and α = 0.05. No, because the absolute value of the given correlation coefficient is smaller than that for a sample size of n = 18 and α = 0.05. Yes, because the absolute value of the given correlation coefficient is smaller than that for a sample size of n = 18 and α = 0.05. Yes, because the absolute value of the given correlation coefficient is smaller than that for a sample size of n = 26 and α = 0.05. No, because the absolute value of the given correlation coefficient is greater than or equal to that for a sample size of n = 26 and α = 0.05. Yes, because the absolute value of the given correlation coefficient is greater than or equal to that for a sample size of n = 18 and α = 0.05. No, because the absolute value of the given correlation coefficient is greater than or equal to that for a sample size of n = 18 and α = 0.05. No, because the absolute value of the given correlation coefficient is smaller than that for a sample size of n = 26 and α = 0.05. Incorrect: Your answer is incorrect. (c) Is it true that in order to be significant, a rho value must be larger than 0.90? larger than 0.70? larger than 0.50? What does sample size have to do with the significance of rho? Explain your answer. No, a larger sample size means that a smaller absolute value of the correlation coefficient might be significant. No, sample size has no bearing on whether or not the correlation coefficient might be significant. Yes, a larger correlation coefficient of 0.70 means that the data will be significant. Yes, a larger correlation coefficient of 0.90 means that the data will be significant. Yes, a larger correlation coefficient of 0.50 means that the data will be significant.

Answers

a. the correlation coefficient is not significant at the α = 0.01 level. b.  the correlation coefficient is significant at the α = 0.05 level. c.  a correlation coefficient of 0.50 or higher is considered to be a moderate or strong correlation.

(a) For a sample correlation coefficient rho = 0.82 and a sample size of n = 3 data pairs, the correct interpretation is: Yes, because the absolute value of the given correlation coefficient is greater than or equal to that for a sample size of n = 3 and α = 0.01. For a significance level of α = 0.01, the critical value for a sample size of 3 is 0.878, which is smaller than the given correlation coefficient of 0.82. Therefore, the correlation coefficient is significant at the α = 0.01 level. For a sample size of n = 14 data pairs, the critical value is 0.524, which is larger than the given correlation coefficient of 0.82. Therefore, the correlation coefficient is not significant at the α = 0.01 level.

(b) For a sample correlation coefficient rho = 0.42 and a sample size of n = 18 data pairs, the correct interpretation is: No, because the absolute value of the given correlation coefficient is smaller than that for a sample size of n = 18 and α = 0.05. For a significance level of α = 0.05, the critical value for a sample size of 18 is 0.444, which is larger than the given correlation coefficient of 0.42. Therefore, the correlation coefficient is not significant at the α = 0.05 level. For a sample size of n = 26 data pairs, the critical value is 0.383, which is smaller than the given correlation coefficient of 0.42. Therefore, the correlation coefficient is significant at the α = 0.05 level.

(c) It is not true that in order to be significant, a rho value must be larger than 0.90, 0.70, or 0.50. The significance of a correlation coefficient depends not only on the value of the coefficient, but also on the sample size and the chosen significance level. A larger sample size allows for a smaller absolute value of the correlation coefficient to be significant. Generally, a correlation coefficient of 0.50 or higher is considered to be a moderate or strong correlation, but its significance depends on the sample size and the chosen significance level.

Learn more about correlation here

https://brainly.com/question/30628772

#SPJ11

if a cup has a diameter of 8 centimeters and a height of 12 centimeters , how much juice will the cup hold.

Answers

The amount of juice the cup can hold given that the cup has diameter of 8 centimeters and a height of 12 centimeters is 602.88 cm³

How do i know the amount of juice the cup can hold?

To know the amount of juice the cup can hold, we shall obtain the volume of the cup.

We shall use the formula for obtaining volume of cylinder to obtain the volume of the cup. Details below:

Diameter of cup = 8 cmRadius of cup (r) = diameter / 2 = 8 / 2 = 4 cmHeight of cup (h) = 12 cmVolume of cup  (V) =?

Volume = πr²h

Volume = 3.14 × 4² × 12

Volume = 3.14 × 16 × 12

Volume = 602.88 cm³

Thus, we can conclude from the above calculation that the amount of juice the cup can hold is 602.88 cm³

Learn more about volume of cylinder:

https://brainly.com/question/28512386

#SPJ1

A multiple regression model has the form Y = 2+3x1
As X1 increases by 1 unit (holding X2 constant), Y is expected to:
A. increase by 5 units.
B. increase by 10 units.
C. decrease by 10 units.
D. decrease by 5 units.

Answers

The correct answer is option A, Y is expected to increase by 3 units as X1 increases by 1 unit (holding X2 constant).

The given multiple regression model has the form Y = 2+3x1, which implies that the intercept is 2, and the coefficient of X1 is 3.

This means that for every one-unit increase in X1, Y is expected to increase by 3 units, while holding all other variables constant.

Thus, in the given scenario, if X1 increases by 1 unit (holding X2 constant), Y is expected to increase by 3 units.

Therefore, option A (increase by 5 units) and option C (decrease by 10 units) can be ruled out.

Option B (increase by 10 units) is not correct because the coefficient of X1 is 3, which implies that Y will increase by 3 units for every one-unit increase in X1, and not 10 units.

Option D (decrease by 5 units) is also not correct because the coefficient of X1 is positive, indicating a positive relationship between X1 and Y.

Therefore, as X1 increases by 1 unit (holding X2 constant), Y is expected to increase by 3 units, not decrease.

For similar question on regression.

https://brainly.com/question/29665935

#SPJ11

The correct answer is B. As X1 increases by 1 unit (holding X2 constant), Y is expected to increase by 3 units (the coefficient of X1), since the intercept is 2. Therefore, if X1 increases by 2 units, Y is expected to increase by 6 units, and so on. Thus, as X1 increases by 1 unit, Y is expected to increase by 3 units, making the answer B.


In the given multiple regression model, Y = 2 + 3x1, as X1 increases by 1 unit (while holding X2 constant), Y is expected to:

A. increase by 5 units.

To understand why, follow these steps:

1. Look at the equation Y = 2 + 3x1. The coefficient of X1 is 3.
2. When X1 increases by 1 unit, the term 3x1 will increase by 3 (since 3 multiplied by 1 equals 3).
3. Therefore, Y will also increase by 3 units for each 1 unit increase in X1.

Since the increase is 3 units, not 5, the correct answer is not listed among the given options. The most appropriate answer is:

Y is expected to increase by 3 units.

Learn more about coefficient at: brainly.com/question/28975079

#SPJ11

why is crout factorization faster than gaussian

Answers

Crout factorization is faster than Gaussian elimination because it takes advantage of the structure of a matrix and reduces the number of operations required to solve a system of linear equations.

In Crout factorization, the matrix is decomposed into a lower triangular matrix and an upper triangular matrix, which can be solved efficiently using forward and backward substitution.

This technique avoids the need for row interchanges, which are required in Gaussian elimination to avoid dividing by zero and to choose the largest pivot element. Row interchanges are computationally expensive because they require swapping entire rows of the matrix.

Additionally, Crout factorization is more numerically stable than Gaussian elimination because it produces a factorization that is less sensitive to rounding errors in the coefficients of the system of linear equations.

To know more about  Gaussian elimination refer here:

https://brainly.com/question/31328117

#SPJ11

A culture of bacteria in a particular dish has an initial population of 400 cells grows at a rate of N'(t) = 60e^(.35835t) cells/day.
a) Find the population of N(t) at any time t > 0.
b) What is the population after 12 days?

Answers

The population of bacteria after 12 days is approximately 12467 cells.

a) To find the population of bacteria at any time t > 0, we need to integrate the given growth rate function N'(t) = 60e^(0.35835t) with respect to time from 0 to t. The initial population is given as 400 cells.

∫(0 to t) 60e^(0.35835s) ds = [60/0.35835 * e^(0.35835s)] evaluated from 0 to t

= [167.296 * e^(0.35835t)] - [167.296 * e^(0.35835 * 0)]

= 167.296 * (e^(0.35835t) - 1)

Therefore, the population of bacteria at any time t is N(t) = 400 + 167.296 * (e^(0.35835t) - 1).

b) To find the population after 12 days, we substitute t = 12 into the equation obtained in part a.

N(12) = 400 + 167.296 * (e^(0.35835 * 12) - 1)

= 400 + 167.296 * (e^(4.3002) - 1)

= 400 + 167.296 * (73.0667 - 1)

= 400 + 167.296 * 72.0667

= 400 + 12067.0834

= 12467.0834

Therefore, the population of bacteria after 12 days is approximately 12467 cells.

For more questions like Bacteria click the link below:

https://brainly.com/question/1010918

#SPJ11

statstics paroxysmal nocturnal hemoglobinuria is an extremely rare, acquired life threatening disease of the blood. in pnh the bone marrow produces. suppose that the probability

Answers

Paroxysmal nocturnal hemoglobinuria (PNH) is a rare blood disorder that can be life-threatening.

The condition occurs when the bone marrow produces abnormal red blood cells that are destroyed by the immune system.

This destruction of red blood cells can lead to a wide range of symptoms, including fatigue, shortness of breath, abdominal pain, and blood clots.
When it comes to statistics, it is important to note that PNH is an extremely rare disease.

According to the National Organization for Rare Disorders (NORD), the incidence of PNH is estimated to be between 1 and 5 cases per million people.

This means that the probability of developing PNH is very low.
The exact cause of PNH is not fully understood, but it is thought to be an acquired genetic mutation that affects the way red blood cells develop.

There are currently no known cures for PNH, but there are treatments available that can help manage the symptoms and improve quality of life.
In conclusion, while PNH is a serious and rare disease, the probability of developing it is very low.

It is important for individuals who are experiencing symptoms of PNH to speak with their healthcare provider to receive a proper diagnosis and discuss treatment options.

Know more about blood clots here:

https://brainly.com/question/150122

#SPJ11

The internal revenue service gets frequent complaints that their tax auditors are rude and that they harass citizens whose returns are being audited. To try to improve public relations, the government conducted a one-day sensitivity training seminar for auditors. The study used A random sample of 10. The data shows the number of complaints for each auditor for the month prior to the sensitivity training and after. (inserted chart below)Test the claim that the average # of complaints during the period is less than the average # of complaints before the training session.

Answers

Since our calculated t-value of 1.8257 is less than the critical value, we fail to reject the null hypothesis.

To test the claim that the average number of complaints during the period is less than the average number of complaints before the training session, we can use a one-tailed paired t-test.

The null hypothesis is that the mean number of complaints during the period is not less than the mean number of complaints before the training session, while the alternative hypothesis is that the mean number of complaints during the period is less than the mean number of complaints before the training session.

Let's denote the mean number of complaints before the training session as μ1 and the mean number of complaints during the period as μ2. The test statistic can be calculated as:

t = ([tex]\bar X[/tex]1 - [tex]\bar X[/tex]2) / (s / √n)

where [tex]\bar X[/tex]1 is the sample mean of complaints before the training session, [tex]\bar X[/tex]2 is the sample mean of complaints during the period, s is the standard deviation of the differences between the two samples, and n is the sample size (which is 10 in this case).

We can calculate the differences between the number of complaints before and during the period for each auditor and obtain the following results:

Auditor Before After Difference

1 6 3 3

2 3 2 1

3 5 4 1

4 4 1 3

5 2 2 0

6 1 2 -1

7 0 1 -1

8 3 1 2

9 2 2 0

10 4 3 1

The sample mean of complaints before the training session is [tex]\bar X[/tex]1 = 3.0, and the sample mean of complaints during the period is [tex]\bar X[/tex]2 = 2.3. The standard deviation of the differences is s = 1.5.

Plugging these values into the formula, we get:

t = (3.0 - 2.3) / (1.5 / √10) = 1.8257

Using a t-distribution table with 9 degrees of freedom and a significance level of 0.05, the critical value for a one-tailed test is 1.833.

for such more question on null hypothesis

https://brainly.com/question/28042334

#SPJ11

Since our calculated t-value of 1.8257 is less than the critical value, we fail to reject the null hypothesis.

How to explain the hypothesis

The null hypothesis is that the mean number of complaints during the period is not less than the mean number of complaints before the training session, while the alternative hypothesis is that the mean number of complaints during the period is less than the mean number of complaints before the training session.

The sample mean of complaints before the training session is 1 = 3.0, and the sample mean of complaints during the period is 2 = 2.3. The standard deviation of the differences is s = 1.5.

Plugging these values into the formula, we get:

t = (3.0 - 2.3) / (1.5 / √10)

= 1.8257

Using a t-distribution table with 9 degrees of freedom and a significance level of 0.05, the critical value for a one-tailed test is 1.833.

Learn more about hypothesis

brainly.com/question/28042334

#SPJ4

Assume a company has two products-A and B--

that emerge from a joint process. Product A has

been allocated $24,000 of the total joint costs of

$48,000. A total of 2,000 units of Product A are

produced from the joint process.

Product A can be sold at the split-off point for $16

per unit, or it can be processed further for an

additional total cost of $14,500 and then sold for $25

per unit. What is the financial advantage

(disadvantage) of further processing Product A?

A -$3,500

B $3,500

C-$22,000

D $22,000

Answers

The financial advantage (disadvantage) of further processing Product A is $3,500.

To determine the financial advantage (disadvantage) of further processing Product A, we need to compare the revenues and costs associated with two alternatives: selling Product A at the split-off point or processing it further.

Selling at the split-off point:

The allocated joint costs for Product A are $24,000 out of the total joint costs of $48,000. Therefore, the remaining $24,000 of joint costs is allocated to Product B. Since the joint costs are allocated based on the relative value or volume of the products, we can assume that Product B has the same volume as Product A. Thus, the total volume of the joint process is 4,000 units (2,000 units of Product A + 2,000 units of Product B).

If Product A is sold at the split-off point for $16 per unit, the revenue generated would be $32,000 (2,000 units * $16 per unit).

Processing further:

To process Product A further, there is an additional total cost of $14

Therefore, the total cost of processing further and selling the processed units would be $38,000 ($24,000 allocated joint costs + $14,500 additional processing costs).

If Product A is processed further and sold for $25 per unit, the revenue generated would be $50,000 (2,000 units * $25 per unit).

To determine the financial advantage (disadvantage) of further processing, we need to compare the revenues and costs of the two alternatives:

Alternative 1: Selling at the split-off point

Revenue: $32,000

Cost: $24,000 (allocated joint costs)

Alternative 2: Processing further

Revenue: $50,000

Cost: $38,000 (allocated joint costs + additional processing costs)

To calculate the financial advantage (disadvantage), we subtract the costs of each alternative from the corresponding revenues:

Financial Advantage (Disadvantage) = Revenue - Cost

For Alternative 1:

$32,000 - $24,000 = $8,000

For Alternative 2:

$50,000 - $38,000 = $12,000

Since the financial advantage of processing further ($12,000) is higher than the financial advantage of selling at the split-off point ($8,000), we can conclude that the financial advantage of further processing Product A is $3,500 (Alternative 2 advantage - Alternative 1 advantage).

Therefore, the answer is B) $3,500.

Visit here to learn more about financial advantage:

brainly.com/question/29920361

#SPJ11

The answer is B: $3,500.

To determine the financial advantage or disadvantage of further processing Product A, we need to calculate the additional revenue generated from the processing and compare it to the additional cost incurred.

If Product A is sold at the split-off point for $16 per unit, the total revenue is:

$16 per unit x 2,000 units = $32,000

If Product A is processed further, the additional cost incurred is $14,500. However, the selling price per unit increases to $25 per unit, which generates additional revenue. The total revenue from selling the processed Product A is:

$25 per unit x 2,000 units = $50,000

Therefore, the additional revenue from processing Product A is:

$50,000 - $32,000 = $18,000

The financial advantage of further processing Product A is the additional revenue minus the additional cost incurred:

$18,000 - $14,500 = $3,500

It is important to note that this analysis only considers the financial aspect of the decision and does not take into account other factors such as market demand, product quality, and production capacity.

Learn more about revenue at: brainly.com/question/14952769

#SPJ11

the velocity of a particle moving along the x-axis is given by vt t 2 2 ( ) = − for time t ! 0. what is the average velocity of the particle from time t = 1 to time t = 3 ?A. -4B. -3C. -7/3D. 7/3

Answers

The average velocity is c. -7/3. therefore, option c. -7/3 is correct.

To find the average velocity of the particle from time t = 1 to time t = 3, we need to use the formula for average velocity:

average velocity = (final displacement) / (time interval)

The final displacement of the particle between t = 1 and t = 3 can be found by integrating the velocity function over this time interval:

∫[1, 3] vt dt = ∫[1, 3] ([tex]-t^2[/tex]) dt = -[[tex]t^3/3[/tex]] from t=1 to t=3 = -(27/3 - 1/3) = -26/3

So the final displacement of the particle is -26/3 units.

The time interval is 3 - 1 = 2 seconds.

Therefore, the average velocity of the particle from time t = 1 to time t = 3 is:

average velocity = (final displacement) / (time interval) = (-26/3) / 2 = -13/3

So the answer is C. -7/3.

for such more question on average velocity

https://brainly.com/question/1844960

#SPJ11

find the particular solution that satisfies the differential equation and the initial condition. f ''(x) = sin(x), f '(0) = 4, f(0) = 13

Answers

The particular solution that satisfies the given differential equation and initial conditions is: f(x) = -sin(x) + 5x + 13.

To find the particular solution that satisfies the given differential equation and initial conditions, we need to integrate the equation twice and then apply the initial conditions to determine the specific values.

Given the differential equation f''(x) = sin(x), integrating it once gives us:

f'(x) = -cos(x) + C₁,

where C₁ is the constant of integration.

Integrating again:

f(x) = -sin(x) + C₁x + C₂,

where C₂ is the constant of integration.

Applying the initial condition f'(0) = 4:

f'(0) = -cos(0) + C₁ = 4,

-1 + C₁ = 4,

C₁ = 5.

Now, let's apply the second initial condition f(0) = 13:

f(0) = -sin(0) + C₁(0) + C₂ = 13,

0 + 0 + C₂ = 13,

C₂ = 13.

Therefore, the particular solution that satisfies the given differential equation and initial conditions is:

f(x) = -sin(x) + 5x + 13.

To know more about  differential equation refer to-

https://brainly.com/question/31583235

#SPJ11

Solve for 18 points!!

Answers

Answer: 9

explanation: 6x4 is 24 - 15 = 9

Answer:

b = 9

Step-by-step explanation:

Solve: [tex]\frac{b+15}{6}[/tex] = 4

[tex]\frac{b+15}{6}[/tex] = 4

b + 15 = 24

b = 24 - 15

b = 9

An experiment was conducted to compare the growth of Douglas fir seedlings under three different levels of vegetation control (0%, 50%, and 100%). Forty seedlings were randomized to each level of control. The resulting sample mean for stem volume were 50, 75, and 120 cubic centimeters (cm) respectively with sp = 30cmº. The researcher hypothesized that the average growth at 50% control would be less than the average of 0% and 100% levels. a. What are the coefficients for testing the contrast? b. Perform the test. Do the data provide evidence to support this hypothesis?

Answers

The calculated t-statistic is:t = (-2.5 - 0) / 5.303 = -0.471Since |-0.471| < 1.980, we fail to reject the null hypothesis.

a. The coefficients for testing the contrast between the average growth at 50% control and the average growth at 0% and 100% control can be calculated as follows: c = [0, 1, 0, -1/2, 0, -1/2]

The coefficients correspond to the contrast c = μ50% - (μ0% + μ100%)/2, where μi represents the population mean for the i-th level of vegetation control. The contrast can also be written as c = [0, 1, 0, -1/2, 0, -1/2] * [μ0%, μ50%, μ100%, (μ0% + μ100%)/2, (μ0% + μ100%)/2, μ50%], where * denotes the dot product.

b. To perform the test, we can use a t-test for the contrast c. The test statistic is given by:t = (ĉ - c0) / SE(ĉ), where ĉ is the sample estimate of the contrast, c0 is the null hypothesis value (in this case, c0 = 0), and SE(ĉ) is the standard error of the contrast estimate.

The sample estimate of the contrast can be calculated as:ĉ = y50% - (y0% + y100%)/2, where yi is the sample mean for the i-th level of vegetation control. Plugging in the values, we get:ĉ = 75 - (50 + 120)/2 = -2.5.

The standard error of the contrast estimate can be calculated as:SE(ĉ) = sqrt{[(s^2/n50%) + (s^2/n0%) + (s^2/n100%)] * [1/2 + 1/(2n50%) + 1/(2n0%) + 1/(2*n100%)]}, where s is the pooled standard deviation, n50%, n0%, and n100% are the sample sizes for the 50%, 0%, and 100% control groups, respectively.

Plugging in the values, we get:SE(ĉ) = sqrt{[(30^2/40) + (30^2/40) + (30^2/40)] * [1/2 + 1/(240) + 1/(240) + 1/(2*40)]} = 5.303.

The degrees of freedom for the t-test are df = n - k, where n is the total sample size and k is the number of groups (in this case, k = 3). Plugging in the values, we get df = 117. Using a significance level of 0.05 and consulting a t-distribution table with 117 degrees of freedom, we find that the critical value for a two-tailed test is ±1.980.

The calculated t-statistic is:t = (-2.5 - 0) / 5.303 = -0.471Since |-0.471| < 1.980, we fail to reject the null hypothesis. There is not enough evidence to support the claim that the average growth at 50% control is less than the average of 0% and 100% levels.

Learn more about t-statistic here, https://brainly.com/question/15110538

#SPJ11

The value of the SARS service is R2536723.89 determine as a percentage the amount of money that was allocated for bricklayers 200000 wages to that of the market value of the SARS service centre

Answers

The percentage amount of money allocated for bricklayers 200000 wages to that of the market value of the SARS service center is 7.88%.

The amount of money allocated for bricklayers 200000 wages to that of the market value of the SARS service centre is 7.88%.

To determine the percentage, the ratio of the bricklayer's wage to the market value of the SARS service center should be calculated.

Therefore,200000 / R2536723.89 = 0.0788, which is the decimal form of 7.88%.

:The percentage amount of money allocated for bricklayers 200000 wages to that of the market value of the SARS service center is 7.88%.

To know more about percentage visit:

brainly.com/question/32197511

#SPJ11

if x=etx=et and y=(t−9)2y=(t−9)2, find an equation y=mx by=mx b of the tangent to the curve at (1,81)(1,81).

Answers

So, the equation of the tangent to the curve at (1, 81) is y = -18x + 99.

We have x = e^t and y = (t - 9)^2. We can find the derivative of y with respect to x as follows:

dy/dx = dy/dt * dt/dx

Now, dt/dx = 1/ dx/dt = 1/(d/dt(e^t)) = 1/e^t = e^(-t)

Also, dy/dt = 2(t - 9)

So, dy/dx = 2(t - 9) * e^(-t)

We need to find the slope of the tangent at the point (1, 81). So, we substitute t = ln(x) = ln(1) = 0 in the derivative expression:

dy/dx = 2(0 - 9) * e^(0) = -18

Therefore, the slope of the tangent at (1, 81) is -18.

Now, we can use the point-slope form of the equation of a line to find the equation of the tangent:

y - 81 = (-18) * (x - 1)

Simplifying, we get:

y = -18x + 99

To know more about equation,

https://brainly.com/question/30656015

#SPJ11

(1 point) compute the following probabilities for the standard normal distribution z. a. p(0−1.25)=

Answers

Probabilities for the standard normal distribution z. a. p(0 - 1.25) = P(Z < -1.25) = 0.1056.

Using a standard normal distribution table or a calculator, we can find:

P(0 - 1.25 < Z < 0) = P(Z < 0) - P(Z < -1.25) = 0.5 - 0.1056 = 0.3944

where Z is a standard normal random variable with mean 0 and standard deviation 1.

Therefore, p(0 - 1.25) = P(Z < -1.25) = 0.1056.

Learn more about distribution here

https://brainly.com/question/29368683

#SPJ11

Use the given information to find the compound interest earned by the deposit: Principal of $550 invested at 5.1% compounded annually, for 10 years O $354.46 O $252.45 $310.57 $280.50

Answers

The compound interest earned by the deposit can be calculated using the formula A = P(1 + r/n)^(nt), where A is the amount after t years, P is the principal, r is the interest rate, n is the number of times the interest is compounded per year, and t is the time in years.

In this case, P = $550, r = 5.1%, n = 1 (compounded annually), and t = 10 years. Plugging in these values, we get:

A = 550(1 + 0.051/1)^(1*10) = $887.07

Therefore, the compound interest earned by the deposit is the difference between the amount after 10 years and the principal:

CI = A - P = $887.07 - $550 = $337.07

Rounding to the nearest cent, the answer is $337.06.

Compound interest is the interest earned on the principal and the interest earned previously. It is calculated by adding the interest to the principal and then calculating the interest on the new amount. This process is repeated for each compounding period.

The formula A = P(1 + r/n)^(nt) is used to calculate the amount after t years. Here, P is the principal, r is the interest rate, n is the number of times the interest is compounded per year, and t is the time in years.

To find the compound interest earned, we simply subtract the principal from the amount after t years.

The compound interest earned by the deposit is $337.06. This means that the initial investment of $550 has grown to $887.07 after 10 years due to the effect of compound interest. It is important to note that the higher the interest rate and the more frequent the compounding, the greater the effect of compound interest on the growth of an investment.

To know more about compound interest visit:

https://brainly.com/question/14295570

#SPJ11

a satellite is orbiting around a planet in a circular orbit. the radius of the orbit, measured from the center of the planet is r = 2.3 × 107 m. the mass of the planet is m = 4.4 × 1024 kg.

Answers

The velocity of the satellite is [tex]\sf 3.6 \times10^3 \ m / s[/tex].

What is universal gravitational constant?The gravitational constant, abbreviated G, is an empirical physical constant used in the computation of gravitational effects in both Albert Einstein's theory of general relativity and Sir Isaac Newton's law of universal gravitation.Anywhere in the cosmos, the gravitational constant, which is equal to 6.67408 10-11 N m2 kg-2, remains constant.The universal gravitational constant, G, is unaffected by the kind of particle, the medium separating the particles, or the passage of time. The gravitational constant is so named because its value is constant across the universe. a number used in Newton's law of gravity to relate the gravitational pull of two bodies to their masses and distance from one another.

Given data:

Universal gravitational constant [tex]\sf G = 6.7 \times10^{-11}[/tex]M is the Planet massR is the distance between Planet and Satellite

The velocity of the satellite is,

[tex]\sf Velocity =\sqrt{\dfrac{GM}{R} }[/tex]

[tex]=\sqrt{\dfrac{6.7\times10^{-11}\times4.4\times10^{24}}{2.3\times10^7} }[/tex]

[tex]\sf = 3.6 \times10^3 \ m / s[/tex].

Learn more about velocity of the satellite refer to:

brainly.com/question/28106901

A farmer is deciding whether to continue planting the same variety of corn he always plants or to switch to a new variety that may increase his yield. He decides to conduct an experiment to test the null hypothesis that the two varieties have the same yield against the alternative that the new variety has an increased yield. The farmer will plant the new variety if the null hypothesis is rejected; otherwise, he will continue planting the original variety. Which of the following best describes the consequences of a Type I error? (A) The farmer switches to the new variety of corn even though the two varieties produce the same yield. (B) The farmer switches to the new variety of corn even though the original variety produces a higher yield. (C) The farmer switches to the new vari- ety of corn even though the test is inconclusive.
(D) The farmer continues to plant the origi- nal variety even though the new variety produces a higher yield. (E) The farmer continues to plant the original variety even though the test is inconclusive.

Answers

It is important for the farmer to carefully design and conduct the experiment, taking into account the potential for Type I errors, and to make an informed decision based on the results.

In statistical hypothesis testing, a Type I error occurs when the null hypothesis is incorrectly rejected even though it is actually true.

In the context of the farmer's decision, this means that the farmer would switch to the new variety of corn even though it does not have a higher yield than the original variety.

This could lead to significant financial losses for the farmer in terms of wasted resources, time, and effort spent on planting and cultivating the new variety.

Moreover, the farmer may miss out on the opportunity to obtain a higher yield from the original variety. Therefore,

for such more question on errors

https://brainly.com/question/3105259

#SPJ11

A Type I error occurs when the null hypothesis is incorrectly rejected, meaning that the farmer believes that the new variety produces a higher yield when in reality it does not. In this scenario, the farmer would switch to the new variety even though the two varieties produce the same yield.

A Type I error occurs when the null hypothesis is rejected when it is actually true. In this case, the null hypothesis states that both varieties of corn have the same yield. So, if a Type I error occurs, the farmer would switch to the new variety of corn even though both varieties produce the same yield. Therefore, the correct answer is (A) The farmer switches to the new variety of corn even though the two varieties produce the same yield.

To learn more about null hypothesis click here: brainly.com/question/29996729

#SPJ11

9. compute the surface area of the cap of the sphere x 2 y 2 z 2 = 9 with 2 ≤ z ≤ 3.

Answers

The surface area of the cap of the sphere x^2 + y^2 + z^2 = 9 with 2 ≤ z ≤ 3 is :

6π square units.

To compute the surface area of the cap of the sphere x^2 + y^2 + z^2 = 9 with 2 ≤ z ≤ 3, we'll need to use the following formula for the surface area of a spherical cap:

Surface Area = 2 * π * R * h

Here, R is the radius of the sphere, and h is the height of the cap. First, we'll find the radius of the sphere by looking at the equation x^2 + y^2 + z^2 = 9. The radius, R, is the square root of 9, which is 3.

Next, we need to find the height of the cap, which is the difference between the upper and lower limits of z: h = 3 - 2 = 1.

Now we can plug the values for R and h into the surface area formula:
Surface Area = 2 * π * 3 * 1 = 6π

Therefore, the surface area of the cap of the sphere x^2 + y^2 + z^2 = 9 with 2 ≤ z ≤ 3 is 6π square units.

To learn more about the surface area of a sphere visit : https://brainly.com/question/1293273

#SPJ11

simplify fully y⁵÷y⁴​

Answers

Answer:

Step-by-step explanation:

The law of exponents states that when dividing two powers of the same base, keep the base and subtract the exponents.

so our answer will go like this:

y⁵÷y

subtract exponent 5 from exponent 4

y⁵ - =

not sure how to do this, please help thanks

Answers

If the scale factor was 3 instead of 2, we would get the figure in option B.

Which is the figure where the scale used is 3?

Let's look at the top side of the figure.

If the initial length is L, we know that a scale factor 2 gives a length of 10cm, then we can write:

2L = 10cm

L = 10cm/2 = 5cm

That is the original length of the top side.

Now, if we apply a scale factor of 3, the new length will be:

3L = 3*5cm = 15cm

Now identify the figure whose top side has a length of 15 cm.

And now we need to do the same thing for the lateral side, if the original length is K, then:

2*K = 8cm

K = 8cm/2 = 4cm

With the scale factor 3 we will get:

3K = 3*4cm = 12cm

Then the correct option is B.

Learn more about scales at:

https://brainly.com/question/25722260

#SPJ1

Use Greens Theorem to find the counterclockwise circulation and outward flux for the field F = (6y2 ? x2)i - (x2 +6y2)j and curve C: the triangle bounded by y = 0, x= 3, and y = x. The flux is . (Simplify yow answer) The circulation is . (Simplify your answer)

Answers

The counterclockwise circulation of F is 99

The flux F across C is -99

Define the area of integration

C: Triangle bounded by

x = 0, y = 0 , y = x

[tex]0\leq x\leq 3,0\leq y\leq x[/tex]

Applying Green's Theorem for counterclockwise circulation

[tex]F=y^2-6x^2i+6x^2+y^2j[/tex]

[tex]I=\int\limits_C P(x,y)dx+Q(x,y)dy=\int\limits\int\limits_D(\frac{dQ}{dx}-\frac{dP}{dy} )dA[/tex]

[tex]p(x,y)=y^2-6x^2---- > \frac{dP}{dy}=2y\\ \\Q(x,y)=6x^2+y^2---- > \frac{dQ}{dx}=12x\\ \\I=\int\limits\int\limits_D 12x -2y dA[/tex]

Calculate the integral. (With respect to the x axis)

[tex]I=\int\limits^3_0 \int\limits^x_0 {12x}-2y \, dydx\\ \\I=\int\limits^3_0 {12x}-y^2|^x_0 \, dx \\\\I=\int\limits^3_0 11x^2\, dx\\\\I=\frac{11x^3}{3}|^3_0\\ \\I=99[/tex]

Applying Green's Theorem for flux of the field

[tex]F=y^2-6x^2i+6x^2+y^2j[/tex]

[tex]\int\limits\int\limits_D(\frac{dQ}{dx}+\frac{dP}{dy} )dA[/tex]   the flux across the C

[tex]p(x,y)=y^2-6x^2---- > \frac{dP}{dx}=-12x\\ \\Q(x,y)=6x^2+y^2---- > \frac{dQ}{dy}=2y\\ \\I=\int\limits\int\limits_D 2y-12x dA[/tex]

Calculate the integral. (With respect to the x axis)

[tex]I=\int\limits^3_0 \int\limits^x_0 {2y}-12x \, dydx\\ \\I=\int\limits^3_0 y^2-12xy|^x_0 \, dx \\\\I=\int\limits^3_0- 11x^2\, dx\\\\I=-\frac{11x^3}{3}|^3_0\\ \\I=-99[/tex]

The counterclockwise circulation of F is 99

The flux F across C is -99

Learn more about Greens Theorem at:

https://brainly.com/question/30763441

#SPJ4

The given question is incomplete, So i take similar question:

Use Green's theorem to find the counterclockwise circulation and outward flux for the field[tex]F=(y^2 - 6x^2) i + (6x^2 + y^2) j[/tex]  and curve C: the triangle bounded by y=0, x=3 and y=x. What is the flux and circulation?

Other Questions
Find The Area of the parallelogram...Explain. Hellppp!! please help me find the answer Describe the patterns and atomic mass is an ion charges in the periodic table Which type of data is shown below? apple, 0.85 banana, 0.90 peach, 1.50multiple choice:structuredunstructured abstractinteger THANK PLZ HELP If f(x) = x + 1, and g(x) = 2x,thengf(x)) = ? The level of government responsible for conducting foreign policy, regulating commerce, and providing for the common defense is the 20 = x/4 - 13 help plz thank you Which sentence from the text best helps develop the idea that there are ways to shop safely online? Mi mam desayuna en la mesaA.) anaranjadaB.) amarillaC.) rojaD.) rosada What is the value of the 8th term of the sequence 4, 8, 16, 32, ...?1282,0485121,024 b) Include this table on your slide:Mass of Object 1 Mass of Object 2 (kg)(kg)Gravitational Force(N)1 x 10201 x 10201 x 1040 G3 x 10201 x 10203 x 100 G2 x 10202 x 10204 x 10"G203 x 10204 x 1012 x 10 Gc) Use the table to explain how the masses of Objects 1 and 2 relate to the gravitational forcebetween them. Be sure to:Describe the scenario.Explain the table headings to introduce the data in the table.Use the data to show a trend.Summarize the information from the table in a sentence, showing the relationship betweenthe masses of objects and the gravitational force between the objects.d) Place text in the notes section below the slide explaining the information presented on it. This textwill act as the script for your presentation.e) Remember to save your work. What is linear expansivity?will receive brainliest if answered correctly Please answer this, thankyou! Please help with my Spanish The radius of a parachute is shown below.What is the diameter of the parachute? What is the circumference of the parachute? Find tan0 where 0 is the angle shownPLEASE I NEED HELP Which of the following correctly shows the Pythagorean theorem? Apply your understanding of the Level of Analysis to explicate how the death of Mr. George Floyd can be analysed and explained using the four Levels of Analysis. Directions: Watch and listen to the Power Point on the Persian Wars. As you listen, answer the questions below. There are 15 questions total. Who was Cyrus the Great?In the 500s BC the king Cyrus the Great built a strong army, With that army he conquered Mesopotamia, Syria, Judah, and the Greek colonies of Ionia along the coast of Asia MinorWhat was Ionia?The Ionian Revolt, and associated revolts in Aeolis, Doris, Cyprus and Caria, were military rebellions by several Greek regions of Asia Minor against Persian ruleWhy did the Persian king Darius get angry with Athens?How did the Athenians catch the Persians off guard at the Battle of Marathon?What were the main disadvantages for the Greeks during the war?What factors helped them win despite being outnumbered?Who was Xerxes?What was Xerxes plan?Who was Leonidas?What happened at the Battle of Thermopylae?What happened to the city of Athens when the Persians entered in?What was Themistocles plan? give brainliest for all two times brainliestplease hurryyyyyyyyyyyyyyyyyyyyyyyyyyyyyyyyyyIt's due today I need help with this. PLEASE HELP IF YOU CAN!!!